Đến nội dung

the unknown

the unknown

Đăng ký: 10-03-2016
Offline Đăng nhập: 15-04-2023 - 15:30
****-

#644400 Tìm $x,y$ nguyên dương để $x^2y^4-y^3+1$ là số chính phương

Gửi bởi the unknown trong 10-07-2016 - 21:01

Tìm $x,y$ nguyên dương để $x^2y^4-y^3+1$ là số chính phương

Đặt $S=x^2y^4-y^3+1$.

Trước hết với trường hợp $y=1$ thì $S=x^2$ là số chính phương với $x$ tùy ý. Do đó cặp số $(x,y)$ thỏa là $(k,1)$ với $k$ là số nguyên dương tùy ý.

Xét $y\geq 2$. Khi đó ta cần có bổ đề sau:

Bổ đề: Với $k$ là số nguyên dương, $k\geq 2$ thì khi đó, nếu $a,b$ là các số nguyên dương thỏa:

$\left\{\begin{matrix} ab=k^3-1\\ k^2\mid a+b\\ \end{matrix}\right.$

thì $(a,b)$ là một hoán vị của $(k^3-1;1)$.

Chứng minh

Trở lại bài toán: Đặt $S=t^2$ suy ra $(xy^2-t)(xy^2+t)=y^3-1$. Nếu đặt $a=xy^2-t$ và $b=xy^2+t$ thì $ab=y^3-1$ và $ y^2\mid a+b$ nên theo bổ đề ta có được $xy^2+t=y^3-1$ và $xy^2-t=1$. Suy ra $2xy^2=y^3\Rightarrow 2x=y$. Suy ra phương trình có nghiệm là $(k,2k)$ với $k$ là số nguyên dương tùy ý, thử lại thấy thỏa.

Kết luận: Phương trình có các nghiệm nguyên là $(k;1)$ và $(k;2k)$ với $k$ nguyên dương tùy ý.




#643772 Tính $j-i$

Gửi bởi the unknown trong 05-07-2016 - 20:23

Số $a_n$ ($n\geq 1$) được lập bằng cách viết liền các số $1,2,3,...,n$ ( ví dụ $a_{12}=123456789101112$). Giả sử $i,j$ là hai số nhỏ nhất ($i< j$) sao cho $11\mid a_i$ và $11\mid a_j$. Tính $j-i$.




#643644 USA TSTST 2016

Gửi bởi the unknown trong 04-07-2016 - 17:41

USA TSTST 2016

(USA Team Selection Test for the Selection Team 2016)

 

Ngày 1 (25/6/2016)

 

Bài 1: Cho hai đa thức $A=A(x,y)$ và $B=B(x,y)$ là hai đa thức hai biến với hệ số thực. Giả sử rằng $A(x,y)/B(x,y)$ là đa thức theo $x$ với vô số giá trị của $y$ và là đa thức theo $y$ với vô số giá trị của $x$. Chứng minh rằng $B | A$, tức là tồn tại một đa thức $C$ với hệ số thực thỏa $A=B.C$

 

Bài 2: Cho $ABC$ là một tam giác không đều với trực tâm $H$ và tâm ngoại tiếp $O$. Gọi $M,N$ là trung điểm của $AH$ và $BC$ tương ứng. Giả sử đường tròn $\gamma$ với đường kính $AH$ cắt $(ABC)$ tại $G\neq A$ và cắt đường thẳng $AN$ tại $Q\neq A$. Tiếp tuyến tại $G$ của $\gamma$ cắt đường thẳng $OM$ tại $P$. Chứng minh rằng giao điểm của hai đường tròn $(GNQ)$ và $(MBC)$ cắt nhau tại điểm $T$ nằm trên $PN$.

 

Bài 3: Có tồn tại hay không một đa thức khác hằng $Q(n)$ với hệ số nguyên thỏa mãn tính chất: Với mọi số nguyên dương $n>2$ thì các số:

$Q(0),Q(1),Q(2),...,Q(n-1)$

nhận không quá $0,499n$ số dư phân biệt theo modulo $n$.

 

Ngày 2 (27/6/2016)

Bài 4: Giả sử $n$ và $k$ là các số nguyên dương thỏa mãn:

$\underbrace{\varphi (\varphi (...\varphi }_{\text{k lần}}(n)...))=1$

Chứng minh rằng $n\leq 3^k$.

Trong đó kí hiệu $\varphi(n)$ là số các số nguyên dương trong tập $\left \{ 1,2,3,...,n \right \}$ mà nguyên tố cùng nhau với $n$.

 

Bài 5: Trong mặt phẳng tọa độ có hữu hạn các bức tường là các đoạn thẳng không giao nhau và không có đoạn nào song song với các trục tọa độ. Một chiếc xe ủi bắt đầu từ một điểm bất kì trên mặt phẳng và di chuyển theo hướng dương của trục hoành. Mỗi khi tông một bức tường, chiếc xe sẽ rẽ một góc vuông và đi xa ra khỏi bức tường đó. ( Vì vậy mà chiếc xe luôn chuyển động song song với các trục tọa độ).

Chứng minh rằng chiếc xe không thể tông cả hai mặt của tất cả các bức tường.

 

Bài 6: Một tam giác $ABC$ có tâm nội tiếp $I$ và đường tròn nội tiếp tiếp xúc với các cạnh $BC,CA,AB$ lần lượt tại các điểm $D,E,F$. Gọi $K$ là chân đường vuông góc hạ từ $D$ xuống $EF$. Giả sử rằng đường tròn ngoại tiếp tam giác $AIB$ gặp đường tròn nội tiếp tại hai điểm phân biệt $C_1,C_2$, và đường tròn ngoại tiếp tam giác $AIC$ gặp đường tròn nội tiếp tại hai điểm phân biệt $B_1,B_2$. Chứng minh rằng trục đẳng phương của $(BB_1B_2)$ và $(CC_1C_2)$ đi qua trung điểm $M$ của $DK$.

 

Nguồn

P.s




#643602 Kĩ thuật đổi biến trong chứng minh bất đẳng thức

Gửi bởi the unknown trong 04-07-2016 - 11:01

Thấy Topic khá hay mà đóng băng cũng gần một tháng rồi nên hơi buồn  :( . Mình xin đóng góp một số bài bất đẳng thức mà kĩ thuật đổi biến là một điều khá cần thiết:

Bài 5: Cho các số thực $a,b,c$ đôi một khác nhau và $k$ là số thực bất kì. Chứng minh:

$\left ( \frac{k+a}{b-c} \right )^2+\left ( \frac{k+b}{c-a} \right )^2+\left ( \frac{k+c}{a-b} \right )^2\geq 2$

Bài 6: Cho $a,b,c$ là các số thực bất kỳ. Chứng minh rằng:

$\left ( \frac{a}{a-kb} \right )^2+\left ( \frac{b}{b-kc} \right )^2+\left ( \frac{c}{c-ka} \right )^2+\frac{2(1-k^3)abc}{(a-kb)(b-kc)(c-kb)}\geq 1$

trong đó $k$ là số thực sao cho $(a-kb)(b-kc)(c-kb)\neq 0$

Bài 7: Cho $a,b,c$ là các số thực đôi một khác nhau. Chứng minh rằng khi đó với mọi $k$, ta luôn có bất đẳng thức:

$\left ( \frac{a+kb}{a-b} \right )^2+\left ( \frac{b+kc}{b-c} \right )^2+\left ( \frac{c+ka}{c-a} \right )^2\geq k^2+1$

Spoiler

Và một kết quả khá nổi tiếng của Vasile Cirtoaje:

Bài 8: Cho $x,y,z$ là các số thực dương thỏa $xyz=1$. Chứng minh rằng:

$\sum \frac{1}{x^2+x+1}\geq 1$




#643560 Số cách để quân cờ không đi qua được bảng

Gửi bởi the unknown trong 04-07-2016 - 08:34

Trên bàn cờ vua, một quân Limp King là một quân cờ có thể di chuyển qua một ô vuông đến bất kì ô nào ở xung quanh nó trừ hai ô ở phía Đông Bắc và phía Tây Nam ( hình minh họa).

p78fws.png

Trên một bàn cờ vua $8\times 8$, một số ô vuông đã bị phá hủy. Để đi qua bảng, quân cờ Limp King phải bắt đầu đứng tại một ô bất kì nào đó chưa bị phá hủy của cạnh phía Bắc của bàn cờ, và thực hiện các bước di chuyển qua các ô vuông chưa bị phá hủy và đi đến một ô vuông chưa bị phá hủy của cạnh phía Nam của bàn cờ. Có bao nhiêu cách để phá hủy chính xác $32$ ô cờ để Limp King không thể đi qua được bảng.

Nguồn

Có thể tham khảo đề gốc:

Spoiler




#643251 Hỏi một chút về Gặp gỡ Toán học 2016 tại Đồng Nai

Gửi bởi the unknown trong 02-07-2016 - 11:03

Mà theo em được biết thì chỉ có các học sinh từ lớp 11 trở lên mới được tham dự thôi. Do đó nếu trường có chọn thì em nghĩ trường sẽ chọn các anh chị lớp trên còn lớp 10 muốn đi thì phải có sự đồng ý của hiệu trưởng. Chỉ tiếc là năm nay tổ chức ở Đồng Nai thì gần nhà em mà năm nay em mới lên lớp 10  :P .

@ Min Nq: Theo mình thì đi thi có sợ "bị 0 điểm" hay không thì không quan trọng. Dĩ nhiên kiến thức có thể mình sẽ không hiểu vì chưa được học nhưng quan trọng là được tham gia trải nghiệm. Còn "nhục" thì mình nghĩ chẳng ai đánh giá bạn đâu vì bạn cũng mới chỉ lớp 10  ^_^ . Một điều quan trọng là bạn sẽ còn có được kỷ yếu GGTH nữa mà, cái đó cũng quý đấy. Nếu bạn có cuốn đó thì sau này có thể cho mình mượn được không? ^_^ Mình cũng muốn tham khảo  ^_^ .




#642791 hệ thặng dư

Gửi bởi the unknown trong 29-06-2016 - 17:06

Chứng minh rằng với mọi n là số nguyên dương luôn tồn tạị $k \in \mathbb{N}$ thỏa mãn:
$$2^{n}|19^{k}-97$$

Bạn có thể tham khảo ở đây: http://diendantoanho...psilon-mathbbn/




#642594 $p+q=r(p-q)^n$

Gửi bởi the unknown trong 28-06-2016 - 10:51

(AoPS) Giả sử rằng $p,q$ là các số nguyên tố sao cho $3\nmid p+q$ và $n,r$ là các số nguyên dương. Tìm tất cả các bộ bốn $(p,q,n,r)$ thỏa mãn $p+q=r(p-q)^n$.




#642436 $x^{4}+4=py^{4}.$

Gửi bởi the unknown trong 27-06-2016 - 14:25

Tìm tất cả các số nguyên tố $p$ sao cho phương trình sau có nghiệm nguyên: $x^{4}+4=py^{4}.$

Bài toán này từng được thảo luận ở đây: http://diendantoanho...-nghiệm-nguyên/




#642325 Inequalities From 2016 Mathematical Olympiads

Gửi bởi the unknown trong 26-06-2016 - 20:10

Bài 46 (JBMO). Cho $a,b,c $ là ba số thực dương. Chứng minh rằng
\[\frac{8}{(a+b)^2 + 4abc} + \frac{8}{(b+c)^2 + 4abc} + \frac{8}{(a+c)^2 + 4abc} + a^2 + b^2 + c ^2 \ge \frac{8}{a+3} + \frac{8}{b+3} + \frac{8}{c+3}.\]
 

Ta có: $\sum \frac{8}{(a+b)^2+4abc}+\sum a^2\geq \sum \frac{8}{(a+b)^2+c(a+b)^2}+\sum \frac{a^2+b^2}{2}\geq \sum (\frac{8}{(c+1)(a+b)^2}+\frac{(a+b)^2}{4})\geq \sum \frac{2\sqrt{2}}{\sqrt{a+1}}$

Bất đẳng thức chứng minh đúng bởi vì ta có đánh giá sau: $\frac{2\sqrt{2}}{\sqrt{a+1}}\geq \frac{8}{a+3}\Leftrightarrow (a+3)^2\geq 8a+8\Leftrightarrow (a-1)^2\geq 0$ ( luôn đúng ).

Và vì vậy nên: $\sum \frac{2\sqrt{2}}{\sqrt{a+1}}\geq \sum \frac{8}{a+3}$.

Vậy bất đẳng thức chứng minh đúng, đẳng thức xảy ra khi và chỉ khi $a=b=c=1$.




#642324 $N=\frac{(a-b)(b-c)(c-a)}{2}+2$

Gửi bởi the unknown trong 26-06-2016 - 20:01

Bài toán: (JBMO 2016) Tìm tất cả bộ ba số nguyên $(a,b,c)$ để số: $N=\frac{(a-b)(b-c)(c-a)}{2}+2$

là một lũy thừa của $2016$.




#642279 MAX: $P=max[a,b,c]-min[a,b,c]$

Gửi bởi the unknown trong 26-06-2016 - 16:44

Cho a,b,c thực thỏa mãn: $a^3+b^3+c^3-3abc=2$ và $a+b+c=2$.

Tìm GTLN của: $P=max[a,b,c]-min[a,b,c]$

Ta có $a^3+b^3+c^3-3abc=(a+b+c)(a^2+b^2+c^2-ab-bc-ca)=2\Rightarrow a^2+b^2+c^2-ab-bc-ca=1$

Như vậy ta có: $\left\{\begin{matrix} a^2+b^2+c^2-ab-bc-ca=1 (1) \\ a+b+c=2\\ \end{matrix}\right.$.

Không mất tính tổng quát có thể giả sử $a=\max[a,b,c]$ và $c=\min[a,b,c]$.

Ta có $b=2-a-c$ và thay vào $(1)$ và biến đổi ta được $a^2+c^2+ac-2a-2b+1=0\Leftrightarrow 4(a^2+c^2+ac-2a-2c+1)=0\Leftrightarrow 3(a+c)^2-8(a+c)+(a-c)^2+4=0\Leftrightarrow 3(a+c-\frac{4}{3})^2+(a-c)^2-\frac{4}{3}=0\geq (a-c)^2-\frac{4}{3}$

Như vậy ta có: $(a-c)^2\leq \frac{4}{3}\Rightarrow a-c\leq \frac{2\sqrt{3}}{3}$.

Đẳng thức xảy ra khi và chỉ khi: $\left\{\begin{matrix} a+c=\frac{4}{3}\\ a-c=\frac{2\sqrt{3}}{3}\\ \end{matrix}\right. \Rightarrow a=\frac{2+\sqrt{3}}{3},b=\frac{2}{3}, c=\frac{2-\sqrt{3}}{3}$.

Do đó GTLN của $\max[a,b,c]-\min[a,b,c]$ là $\frac{2\sqrt{3}}{3}$, xảy ra khi $a=\frac{2+\sqrt{3}}{3},b=\frac{2}{3}, c=\frac{2-\sqrt{3}}{3}$ và các hoán vị.




#642095 $x^5+y^5+1=(x+2)^5+(y-3)^5$

Gửi bởi the unknown trong 25-06-2016 - 10:14

 

P/S: Có lẽ đề sai. 

Thực ra đề không sai đâu bạn, chỉ là đề gốc là chứng minh không tồn tại các số nguyên $x,y$ thôi  :D .

Có một cách làm khác sử dụng định lí Fermat nhỏ:

Ta có: $x^5+y^5+1\equiv x+y+1(\mod 5)$ và $(x+2)^5+(y-3)^5\equiv x+2+y-3=x+y-1 (\mod 5)$

Và như vậy ta có: $x+y+1\equiv x+y-1(\mod5)\Rightarrow 2\equiv 0(\mod5)$ ( vô lí) nên phương trình vô nghiệm.

 

Một bài toán khác: Giải phương trình nghiệm nguyên: $x^5+y^5-1=(x+2)^5+(y-3)^5$.

p.s




#641732 MAX: $M=\frac{1}{1+x^2}+\frac{1}...

Gửi bởi the unknown trong 22-06-2016 - 14:43

Cho x,y,z dương thỏa mãn: $xy+yz+zx=1$.

Tìm MAX của: $M=\frac{1}{1+x^2}+\frac{1}{1+y^2}+\frac{1}{1+z^2}$

Để ý rằng ta có bất đẳng thức quen thuộc: $8(x+y+z)(xy+yz+zx)\leq9(x+y)(y+z)(z+x)$. Thật vậy bất đẳng thức này tương đương với $\sum x(y-z)^2\geq 0$ luôn đúng do $x,y,z$ dương, kết hợp với giả thiết $xy+yz+zx=1$ ta có: $\frac{x+y+z}{(x+y)(y+z)(z+x)}\leq \frac{9}{8}$,

Như vậy ta có: $\sum \frac{1}{x^2+1}= \sum \frac{1}{x^2+xy+xz+yz}=\sum \frac{1}{(x+z)(x+y)}=\frac{2(x+y+z)}{(x+y)(y+z)(z+x)}\leq 2.\frac{9}{8}=\frac{9}{4}$.

Đẳng thức xảy ra khi và chỉ khi $x=y=z=\frac{1}{\sqrt{3}}$.




#641027 $\left\{\begin{matrix} x^{3}+x(y...

Gửi bởi the unknown trong 18-06-2016 - 11:09

Giải hệ phương trình: $$\left\{\begin{matrix} x^{3}+x(y-z)^{2}=2\\y^{3}+y(z-x)^{2}=30 \\z^{3}+z(x-y)^{2}=16 \end{matrix}\right.$$

Từ hệ phương trình ta có: $x^3+x(y-z)^2+y^3+y(x-z)^2=2(z^3+z(x-y)^2)\Leftrightarrow (x+y-2z)(x^2+y^2+z^2)=0$. Hơn nữa nếu $x^2+y^2+z^2=0$ thì $x=y=z=0$ không thỏa mãn hệ. Vậy $x+y-2z=0$ hay $z=\frac{x+y}{2}$ và thay vào hai phương trình đầu ta có được:

$\left\{\begin{matrix} x^3+x\frac{(x-y)^2}{4}=2\\ y^3+y\frac{(x-y)^2}{4}=30\\ \end{matrix}\right. \Rightarrow \left\{\begin{matrix} 4x^3+x(x-y)^2=8\\ 4y^3+y(x-y)^2=120\\ \end{matrix}\right. \Rightarrow 15(4x^3+x(x-y)^2)-(4y^3+y(x-y)^2)=0\Leftrightarrow (3x-y)(25x^2-2xy+5y^2)=0$.

Nếu $25x^2-2xy+5y^2=0$ suy ra $x=y=0$ nên vô lí.

Nếu $3x-y=0$ thì $4x^3+x(x-y)^2=8x^3=8\Rightarrow x=1\Rightarrow y=3\Rightarrow z=2$.

Vậy hệ phương trình có nghiệm duy nhất $(1;3;2)$.